Difference between revisions of "2019 AIME II Problems/Problem 15"

m (Problem)
m (Solution)
Line 3: Line 3:
  
 
==Solution==
 
==Solution==
 +
Let <math>AP=a, AQ=b, cos\angle A = k</math>
 +
 +
Therefore <math>AB= \frac{b}{k} , AC= \frac{a}{k}
 +
 +
By power of point, we have
 +
</math>AP*BP=XP*YP , AQ*CQ=YQ*XQ<math>
 +
Which are simplified to
 +
</math>400= \frac{ab}{k} - a^2<math>
 +
</math>525= \frac{ab}{k} - b^2<math>
 +
Or
 +
</math>a^2= \frac{ab}{k} - 400<math>
 +
</math>b^2= \frac{ab}{k} - 525<math>
 +
(1)
 +
 +
Or
 +
</math>k= \frac{ab}{a^2+400} = \frac{ab}{b^2+525}
 +
 +
Let <math>u=a^2+400=b^2+525</math>
 +
Then, <math>a=\sqrt{u-400},b=\sqrt{u-525},k=\frac{\sqrt{(u-400)(u-525)}}{u}
 +
 +
 +
In triangle </math>APQ<math>, by law of cosine
 +
 +
</math>25^2= a^2 + b^2 - 2abk<math>
 +
 +
Pluging (1)
 +
 +
</math>625=  \frac{ab}{k} - 400 + \frac{ab}{k} - 525 -2abk<math>
 +
 +
Or
 +
 +
</math> \frac{ab}{k} - abk =775<math>
 +
 +
Substitute everything by </math>u<math>
 +
 +
</math>u- \frac{(u-400)(u-525)}{u} =775<math>
 +
 +
The quadratic term is cancelled out after simplified
 +
 +
Which gives </math>u=1400<math>
 +
 +
Plug back in, </math>a= \sqrt{1000} , b=\sqrt{775}<math>
 +
 +
Then
 +
 +
</math>AB*AC= \frac{a}{k} \frac{b}{k} = \frac{ab}{\frac{ab}{u} *\frac{ab}{u} } = \frac{u^2}{ab}
 +
= \frac{1400 * 1400}{ \sqrt{ 1000* 875 }} = 560 \sqrt{14}
 +
 +
So the final answer is 560 + 14 = 574
  
 
==See Also==
 
==See Also==
 
{{AIME box|year=2019|n=II|num-b=14|after=Last Question}}
 
{{AIME box|year=2019|n=II|num-b=14|after=Last Question}}
 
{{MAA Notice}}
 
{{MAA Notice}}

Revision as of 20:14, 22 March 2019

Problem

In acute triangle $ABC$ points $P$ and $Q$ are the feet of the perpendiculars from $C$ to $\overline{AB}$ and from $B$ to $\overline{AC}$, respectively. Line $PQ$ intersects the circumcircle of $\triangle ABC$ in two distinct points, $X$ and $Y$. Suppose $XP=10$, $PQ=25$, and $QY=15$. The value of $AB\cdot AC$ can be written in the form $m\sqrt n$ where $m$ and $n$ are positive relatively prime integers. Find $m+n$.

Solution

Let $AP=a, AQ=b, cos\angle A = k$

Therefore $AB= \frac{b}{k} , AC= \frac{a}{k}

By power of point, we have$ (Error compiling LaTeX. Unknown error_msg)AP*BP=XP*YP , AQ*CQ=YQ*XQ$Which are simplified to$400= \frac{ab}{k} - a^2$$ (Error compiling LaTeX. Unknown error_msg)525= \frac{ab}{k} - b^2$Or$a^2= \frac{ab}{k} - 400$$ (Error compiling LaTeX. Unknown error_msg)b^2= \frac{ab}{k} - 525$(1)

Or$ (Error compiling LaTeX. Unknown error_msg)k= \frac{ab}{a^2+400} = \frac{ab}{b^2+525}

Let $u=a^2+400=b^2+525$ Then, $a=\sqrt{u-400},b=\sqrt{u-525},k=\frac{\sqrt{(u-400)(u-525)}}{u}


In triangle$ (Error compiling LaTeX. Unknown error_msg)APQ$, by law of cosine$25^2= a^2 + b^2 - 2abk$Pluging (1)$625= \frac{ab}{k} - 400 + \frac{ab}{k} - 525 -2abk$Or$ \frac{ab}{k} - abk =775$Substitute everything by$u$$ (Error compiling LaTeX. Unknown error_msg)u- \frac{(u-400)(u-525)}{u} =775$The quadratic term is cancelled out after simplified

Which gives$ (Error compiling LaTeX. Unknown error_msg)u=1400$Plug back in,$a= \sqrt{1000} , b=\sqrt{775}$Then$AB*AC= \frac{a}{k} \frac{b}{k} = \frac{ab}{\frac{ab}{u} *\frac{ab}{u} } = \frac{u^2}{ab} = \frac{1400 * 1400}{ \sqrt{ 1000* 875 }} = 560 \sqrt{14}

So the final answer is 560 + 14 = 574

See Also

2019 AIME II (ProblemsAnswer KeyResources)
Preceded by
Problem 14
Followed by
Last Question
1 2 3 4 5 6 7 8 9 10 11 12 13 14 15
All AIME Problems and Solutions

The problems on this page are copyrighted by the Mathematical Association of America's American Mathematics Competitions. AMC logo.png